ON THI TNPT 2017 24 08 ma de

24 691 1
ON THI TNPT 2017 24 08 ma de

Đang tải... (xem toàn văn)

Tài liệu hạn chế xem trước, để xem đầy đủ mời bạn chọn Tải xuống

Thông tin tài liệu

SỞ GIÁO DỤC ĐÀO TẠO ĐỀ CHÍNH THỨC (Đề gồm có 03 trang) ÔN THI TỐT NGHIỆP TRUNG HỌC PHỔ THÔNG NĂM HỌC 2016- 2017 MÔN TIẾNG ANH ~ ĐỀ 444 Thời gian: 60 phút - không tính thời gian giao đề Mark(s) Phách ……… Mark the letter A, B, C, or D on your answer sheet to indicate the correct answer to each of the following questions Question 1: The man said that he her the day before A met B had met C will meet D have met Question 2: It was in 1989 the Berlin Wall collapsed A that B which C when D what Question 3: Marie Curie was the first and only woman .two Nobel prizes A to have won B to be won C who win D that win Question 4: What are the .between women in old time and women in modem time? A difference B differs C differences D different Question 5: .you study for these exams .you will A The hardest - the best B The harder - the better C The more hard - the more good D The more - the much Question 6: .it with my own eyes, I would never have believed it A If I had seen B Unless I had not seen C Provided I had seen D Had I not seen Question 7: I remember .him a message yesterday A sends B sent C to send D sending Question 8: Nam never comes to class on time and A so doesn’t Hung B so does Hung C neither does Hung D neither doesn’t Hung Question 9: I .an old friend of mine in the street this morning We haven’t seen each other for ages A ran out B came over C came round D ran into Question 10: Sorry, I’m late, but my car on the way here, and I had to call the garage A broke down B out of order C was broke D not working Question 11: Charles was wearing at the party A a tie yellow silk funny B a funny wide yellow silk tie C very funny wide yellow silk tie D a yellow silk funny tie Question 12: that he couldn’t go on working A Such was his pain B So was his pain C His pain was such D He has so pain Read the following passage and mark, the letter A, B, C, or D on your answer sheet to indicate the correct answer to each of the questions from 13 to 20 After inventing dynamite, Swedish-bom Alfred Nobel became a very rich man However, he foresaw its universally destructive powers too late Nobel preferred not to be remembered as the inventor of dynamite, so in 1895, just two weeks before his death, he created a fund to be used for awarding prizes to people who had made worthwhile contributions to humanity Originally, there were five awards: literature, physics, chemistry, medicine, and peace Economics was added in 1968, just sixty-seven years after the first awards ceremony Nobel's original legacy of nine million dollars was invested, and the interest on this sum is used for the awards which vary from $30,000 to $125,000 Every year on December 10, the Einniversary of Nobel's death, the awsirds (gold medal, illuminated diploma, and money) are presented to the winners Sometimes politics plays an important role in the judges' decisions Americans have won numerous science awards, but relatively few literature prizes No awards were presented from 1940 to 1942 at the beginning of World War II Some people have won two prizes, but this is rare; others have shared their prizes Question 13: What is the main idea of this passage? A Alfred Nobel created awards in six categories for contributions to humanity B Alfred Nobel made a lasting contribution to humanity C Alfred Nobel became very rich when he invented dynamite D Alfred Nobel left all of his money to science Question 14: The word "foresaw" in the first paragraph is nearest in meaning to A predicted B postponed C prevented D prevailed Question 15: All of the following statements are true EXCEPT A A few individuals have won two awards B Ceremonies are held on December 10 to commemorate Nobel's invention C Awards vary in monetary value D Politics plays an important role in selecting the winners Question 16: In which area have Americans received the most awards? A Science B Peace C Economics D Literature Question 17: It is implied that Nobel's profession was in A medicine B literature C science D economics Question 18: In the first paragraph, "worthwhile" is closest in meaning to A trivial B prestigious C economic D valuable Question 19: The Nobel prize was established in order to A resolve political differences B spend money C recognize worthwhile contributions to humanity D honour the inventor of dynamite Question 20: The word "legacy" in the second paragraph means most nearly the same as A bequest B debt C prize D legend Mark the letter A, B, C, or D on your answer sheet to indicate the underlined part that needs correction in each of the following questions Question 21: The room is so dirty that it needs to clean without delay A The B delay C so D to clean Question 22: The package must be wrapped careful before it is posted A wrapped careful B is posted C must D The package Question 23: Nowhere you can enioy delicious dishes like here A you can B like C Nowhere D dishes Read the following passage and mark the letter A, B, C, or D on your answer sheet to indicate the correct answer to each of the questions from 24 to 30 Before applying to a foreign university, one should consider the disadvantages of studying abroad First, a student may feel lonely by being far home and friends Also, difficulty in understanding a foreign language can be very disappointing and can affect the student’s grades It can be very expensive to pay the costs of travel and housing in a different country Finally, if there is an emergency at home, it will be hard to get home in a hurry However, studying in another country is advantageous in many ways A student is exposed to a new culture This culture teaches him or her about other people and other way of thinking, which can be promote friendships among countries He can learn an new language which may be beneficial for keeping up with research after student has finished studying He can learn experience not available in his country The student may also get the opportunity to study at a university where leading expert in his field may be teaching The leading expert can introduce the student to the most up-to-date finding of the tops researches in the field Exposure to such available knowledge and insights into the field can aid the student in becoming an expert as well Question 24: .to study abroad A There are only disadvantages B There are only advantages C There are neither advantages nor advantages D There are both advantages and disadvantages Question 25: When studying abroad, a student is exposed to a new culture that A enriches his knowledge about people and other ways of thinking B has no advantage C causes some troubles D has no effects Question 26: At a foreign university, a student may A All are correct B meet his neighbour C meet a leading expert in his field D travel a lot Question 27: The word “available” in the second paragraph refers to A ready B domestic C difficult D mischievous Question 28: At a foreign university, a student A has to learn his mother tongue B can learn a foreign language C cannot learn a foreign language D doesn’t have to learn a language Question 29: The word “affect” in the first paragraph refers to A inform B insist C invest D impact Question 30: When studying abroad, a student A is homesick and feel lonely B cannot go home immediately in emergency C spends a lot of money for accommodation and transportation D All are correct Mark the letter A, B, C, or D on your answer sheet to indicate the most suitable response to complete each of the following exchanges Question 31: ~ Tim: " " ~ John: “Never mind.” A Congratulations! How wonderful! B Would you mind going to dinner next Sunday? C Sorry for staining your carpet Let me have it cleaned D Thank you for being honest with me Question 32: ~ Nam: "May I take my break now?" ~ Huy: " " A Yes, of course B No, you didn’t break it C I think I need five minutes D I’ll be back in five minutes Mark the letter A, B, C, or D on your answer sheet to indicate the word whose underlined part differs from the other three in pronunciation in each of the following questions Question 33:A leisure B measure C pleasure D pressure Question 34:A completed B defended C composed D supported Mark the letter A, B, C, or D on your answer sheet to indicate the word(s) OPPOSITE in meaning to the underlined word(s) in each of the following questions Question 35: My little daughter would spend an inordinate amount of time in the shop, deciding which goods she was going to buy A excessive B limited C abundant D required Question 36: Because Jack defaulted on his loan, the bank took him to court A had a bad personality B was paid much money C paid in full D failed to pay Mark the letter A, B, C, or D on your answer sheet to indicate the sentence that best combines each pair of sentences in the following questions Question 37: They're my two sisters They aren't teachers like me A Like me, neither of my two sisters aren't teachers B They're my two sisters, neither of whom are teachers like me C They're my two sisters, both of those are teachers like me D They're my two sisters who neither are teachers like me Question 38: I did not read his book I did not understand what the lecturer was saying A I found it very difficult to understand what the lecturer said in his book B I would have understood what the lecturer was saying ifI had read his book C What the lecturer wrote and said was too difficult for me to understand D The lecturer's book which I had not read was difficult to understand Mark the letter A, B, C, or D on your answer sheet to indicate the sentence that is closest in meaning to each of the following questions Question 39: You can refuse to answer my question in no way A In no way you can refuse to answer my question B In no way can you refuse to answer my question C In no way you refuse to answer my question D In no way you refuse to answer my question Question 40: People believe that he is a kind doctor A He is loved by people because he is a kind doctor B Everybody admires him because he is a kind doctor C He is believed to be a kind doctor D He is really a kind doctor Question 41: We lost the game because of my mistakes A Although I made mistakes, we didn’t lose the game B Despite my mistakes, we didn’t lose the game C I made mistakes but we lost the game D If I hadn ’t made mistakes, we wouldn’t have lost the game Mark the letter A, B, C, or D on your answer sheet to indicate the word(s) CLOSEST in meaning to the underlined word(s) in each of the following questions Question 42: Dozens of applicants showed up for the vacant position, but only a handful of them were shortlisted for the interview A hand B class C small number D small amount Question 43: I could see the finish line and thought I was home and dry A unsuccessful B hopeless C successful D hopeful Read the following passage and mark the letter A, B, C, or D on your answer sheet to indicate the correct word or phrase that best fits each of the numbered blanks from 45 to 50 Water is necessary for life People can live only a few days (44) it Yet nearly 25 million people die each year because of it Both industrial nations and developing countries are worried about the (45) and quantity of water in the world Even though people, animals, agriculture, and industry use a lot of water, there is more than enough on the earth Water covers about three-quarters of the Earth’s surface However, 97.4 percent of it is salt water Three-fourths of the Earth’s fresh water is frozen in glaciers and in the great polar ice caps Most of the water we use (46) from rivers, lakes, and the atmosphere Less than one percent of the Earth’s water is usable, and we use it over and over again One of the (47) about water is distribution Water is not always distributed where the large (48) centres are Some regions get enough rain But it is all in one or two short rainy seasons Question 44:A without B for C in D with Question 45:A situation B characteristics C quality D conditions Question 46:A arrives B comes C goes D gets Question 47:A facts B things C problems D cases Question 48:A population B men C people D women Mark the letter A, B, C, or D on your answer sheet to indicate the word that differs from the other three in the position of primary stress in each of the following questions Question 49:A wonderful B publisher C character D understand Question 50:A commitment B inflation C enterprise D collection SỞ GIÁO DỤC ĐÀO TẠO ĐỀ CHÍNH THỨC (Đề gồm có 03 trang) ÔN THI TỐT NGHIỆP TRUNG HỌC PHỔ THÔNG NĂM HỌC 2016- 2017 MÔN TIẾNG ANH ~ ĐỀ 542 Thời gian: 60 phút - không tính thời gian giao đề Mark(s) Phách ……… Mark the letter A, B, C, or D on your answer sheet to indicate the word whose underlined part differs from the other three in pronunciation in each of the following questions Question 1:A pleasure B leisure C measure D pressure Question 2:A composed B defended C supported D completed Mark the letter A, B, C, or D on your answer sheet to indicate the most suitable response to complete each of the following exchanges Question 3: ~ Tim: " " ~ John: “Never mind.” A Would you mind going to dinner next Sunday? B Thank you for being honest with me C Sorry for staining your carpet Let me have it cleaned D Congratulations! How wonderful! Question 4: ~ Nam: "May I take my break now?" ~ Huy: " " A I’ll be back in five minutes B No, you didn’t break it C I think I need five minutes D Yes, of course Mark the letter A, B, C, or D on your answer sheet to indicate the word that differs from the other three in the position of primary stress in each of the following questions Question 5:A understand B character C wonderful D publisher Question 6:A enterprise B inflation C collection D commitment Mark the letter A, B, C, or D on your answer sheet to indicate the correct answer to each of the following questions Question 7: .you study for these exams .you will A The more - the much B The hardest - the best C The harder - the better D The more hard - the more good Question 8: .it with my own eyes, I would never have believed it A Unless I had not seen B Had I not seen C If I had seen D Provided I had seen Question 9: Marie Curie was the first and only woman .two Nobel prizes A to have won B who win C that win D to be won Question 10: The man said that he her the day before A met B had met C have met D will meet Question 11: I .an old friend of mine in the street this morning We haven’t seen each other for ages A came round B ran into C came over D ran out Question 12: Sorry, I’m late, but my car on the way here, and I had to call the garage A broke down B was broke C out of order D not working Question 13: that he couldn’t go on working A He has so pain B Such was his pain C So was his pain D His pain was such Question 14: I remember .him a message yesterday A sends B to send C sent D sending Question 15: What are the .between women in old time and women in modem time? A different B difference C differs D differences Question 16: Nam never comes to class on time and A so doesn’t Hung B so does Hung C neither does Hung D neither doesn’t Hung Question 17: It was in 1989 the Berlin Wall collapsed A what B that C when D which Question 18: Charles was wearing at the party A a tie yellow silk funny B very funny wide yellow silk tie C a yellow silk funny tie D a funny wide yellow silk tie Mark the letter A, B, C, or D on your answer sheet to indicate the sentence that best combines each pair of sentences in the following questions Question 19: They're my two sisters They aren't teachers like me A Like me, neither of my two sisters aren't teachers B They're my two sisters, both of those are teachers like me C They're my two sisters who neither are teachers like me D They're my two sisters, neither of whom are teachers like me Question 20: I did not read his book I did not understand what the lecturer was saying A I would have understood what the lecturer was saying ifI had read his book B I found it very difficult to understand what the lecturer said in his book C The lecturer's book which I had not read was difficult to understand D What the lecturer wrote and said was too difficult for me to understand Mark the letter A, B, C, or D on your answer sheet to indicate the word(s) CLOSEST in meaning to the underlined word(s) in each of the following questions Question 21: I could see the finish line and thought I was home and dry A hopeless B unsuccessful C successful D hopeful Question 22: Dozens of applicants showed up for the vacant position, but only a handful of them were shortlisted for the interview A small amount B class C hand D small number Mark the letter A, B, C, or D on your answer sheet to indicate the word(s) OPPOSITE in meaning to the underlined word(s) in each of the following questions Question 23: My little daughter would spend an inordinate amount of time in the shop, deciding which goods she was going to buy A abundant B excessive C required D limited Question 24: Because Jack defaulted on his loan, the bank took him to court A had a bad personality B failed to pay C was paid much money D paid in full Read the following passage and mark the letter A, B, C, or D on your answer sheet to indicate the correct answer to each of the questions from 25 to 31 Before applying to a foreign university, one should consider the disadvantages of studying abroad First, a student may feel lonely by being far home and friends Also, difficulty in understanding a foreign language can be very disappointing and can affect the student’s grades It can be very expensive to pay the costs of travel and housing in a different country Finally, if there is an emergency at home, it will be hard to get home in a hurry However, studying in another country is advantageous in many ways A student is exposed to a new culture This culture teaches him or her about other people and other way of thinking, which can be promote friendships among countries He can learn an new language which may be beneficial for keeping up with research after student has finished studying He can learn experience not available in his country The student may also get the opportunity to study at a university where leading expert in his field may be teaching The leading expert can introduce the student to the most up-to-date finding of the tops researches in the field Exposure to such available knowledge and insights into the field can aid the student in becoming an expert as well Question 25: When studying abroad, a student A cannot go home immediately in emergency B All are correct C is homesick and feel lonely D spends a lot of money for accommodation and transportation Question 26: At a foreign university, a student A can learn a foreign language B has to learn his mother tongue C doesn’t have to learn a language D cannot learn a foreign language Question 27: .to study abroad A There are both advantages and disadvantages B There are only advantages C There are neither advantages nor advantages D There are only disadvantages Question 28: The word “available” in the second paragraph refers to A domestic B difficult C mischievous D ready Question 29: At a foreign university, a student may A meet his neighbour B All are correct C travel a lot D meet a leading expert in his field Question 30: The word “affect” in the first paragraph refers to A impact B inform C invest D insist Question 31: When studying abroad, a student is exposed to a new culture that A causes some troubles B has no effects C enriches his knowledge about people and other ways of thinking D has no advantage Mark the letter A, B, C, or D on your answer sheet to indicate the underlined part that needs correction in each of the following questions Question 32: The room is so dirty that it needs to clean without delay A to clean B delay C so D The Question 33: The package must be wrapped careful before it is posted A wrapped careful B is posted C must D The package Question 34: Nowhere you can enioy delicious dishes like here A you can B Nowhere C like D dishes Read the following passage and mark the letter A, B, C, or D on your answer sheet to indicate the correct word or phrase that best fits each of the numbered blanks from 35 to 39 Water is necessary for life People can live only a few days (35) it Yet nearly 25 million people die each year because of it Both industrial nations and developing countries are worried about the (36) and quantity of water in the world Even though people, animals, agriculture, and industry use a lot of water, there is more than enough on the earth Water covers about three-quarters of the Earth’s surface However, 97.4 percent of it is salt water Three-fourths of the Earth’s fresh water is frozen in glaciers and in the great polar ice caps Most of the water we use (37) from rivers, lakes, and the atmosphere Less than one percent of the Earth’s water is usable, and we use it over and over again One of the (38) about water is distribution Water is not always distributed where the large (39) centres are Some regions get enough rain But it is all in one or two short rainy seasons Question 35:A with B for C in D without Question 36:A situation B quality C characteristics D conditions Question 37:A comes B arrives C goes D gets Question 38:A facts B things C problems D cases Question 39:A people B population C men D women Mark the letter A, B, C, or D on your answer sheet to indicate the sentence that is closest in meaning to each of the following questions Question 40: You can refuse to answer my question in no way A In no way you refuse to answer my question B In no way you refuse to answer my question C In no way you can refuse to answer my question D In no way can you refuse to answer my question Question 41: We lost the game because of my mistakes A If I hadn ’t made mistakes, we wouldn’t have lost the game B Despite my mistakes, we didn’t lose the game C I made mistakes but we lost the game D Although I made mistakes, we didn’t lose the game Question 42: People believe that he is a kind doctor A He is really a kind doctor B He is believed to be a kind doctor C He is loved by people because he is a kind doctor D Everybody admires him because he is a kind doctor Read the following passage and mark, the letter A, B, C, or D on your answer sheet to indicate the correct answer to each of the questions from 43 to 50 After inventing dynamite, Swedish-bom Alfred Nobel became a very rich man However, he foresaw its universally destructive powers too late Nobel preferred not to be remembered as the inventor of dynamite, so in 1895, just two weeks before his death, he created a fund to be used for awarding prizes to people who had made worthwhile contributions to humanity Originally, there were five awards: literature, physics, chemistry, medicine, and peace Economics was added in 1968, just sixty-seven years after the first awards ceremony Nobel's original legacy of nine million dollars was invested, and the interest on this sum is used for the awards which vary from $30,000 to $125,000 Every year on December 10, the Einniversary of Nobel's death, the awsirds (gold medal, illuminated diploma, and money) are presented to the winners Sometimes politics plays an important role in the judges' decisions Americans have won numerous science awards, but relatively few literature prizes No awards were presented from 1940 to 1942 at the beginning of World War II Some people have won two prizes, but this is rare; others have shared their prizes Question 43: It is implied that Nobel's profession was in A medicine B economics C literature D science Question 44: The word "legacy" in the second paragraph means most nearly the same as A debt B bequest C legend D prize Question 45: The Nobel prize was established in order to A spend money B honour the inventor of dynamite C recognize worthwhile contributions to humanity D resolve political differences Question 46: The word "foresaw" in the first paragraph is nearest in meaning to A postponed B prevailed C prevented D predicted Question 47: What is the main idea of this passage? A Alfred Nobel left all of his money to science B Alfred Nobel created awards in six categories for contributions to humanity C Alfred Nobel made a lasting contribution to humanity D Alfred Nobel became very rich when he invented dynamite Question 48: In the first paragraph, "worthwhile" is closest in meaning to A economic B prestigious C valuable D trivial Question 49: All of the following statements are true EXCEPT A Awards vary in monetary value B Ceremonies are held on December 10 to commemorate Nobel's invention C Politics plays an important role in selecting the winners D A few individuals have won two awards Question 50: In which area have Americans received the most awards? A Science B Economics C Literature D Peace SỞ GIÁO DỤC ĐÀO TẠO ĐỀ CHÍNH THỨC (Đề gồm có 00 trang) ÔN THI TỐT NGHIỆP TRUNG HỌC PHỔ THÔNG NĂM HỌC 2016- 2017 MÔN TIẾNG ANH ~ ĐỀ 392 Thời gian: 60 phút - không tính thời gian giao đề Mark(s) Phách ……… Mark the letter A, B, C, or D on your answer sheet to indicate the word whose underlined part differs from the other three in pronunciation in each of the following questions Question 1:A measure B leisure C pressure D pleasure Question 2:A supported B composed C completed D defended Mark the letter A, B, C, or D on your answer sheet to indicate the word that differs from the other three in the position of primary stress in each of the following questions Question 3:A enterprise B inflation C commitment D collection Question 4:A understand B publisher C character D wonderful Mark the letter A, B, C, or D on your answer sheet to indicate the most suitable response to complete each of the following exchanges Question 5: ~ Nam: "May I take my break now?" ~ Huy: " " A Yes, of course B I’ll be back in five minutes C I think I need five minutes D No, you didn’t break it Question 6: ~ Tim: " " ~ John: “Never mind.” A Would you mind going to dinner next Sunday? B Sorry for staining your carpet Let me have it cleaned C Congratulations! How wonderful! D Thank you for being honest with me Mark the letter A, B, C, or D on your answer sheet to indicate the sentence that is closest in meaning to each of the following questions Question 7: We lost the game because of my mistakes A I made mistakes but we lost the game B If I hadn ’t made mistakes, we wouldn’t have lost the game C Despite my mistakes, we didn’t lose the game D Although I made mistakes, we didn’t lose the game Question 8: You can refuse to answer my question in no way A In no way you refuse to answer my question B In no way you refuse to answer my question C In no way you can refuse to answer my question D In no way can you refuse to answer my question Question 9: People believe that he is a kind doctor A He is loved by people because he is a kind doctor B He is really a kind doctor C Everybody admires him because he is a kind doctor D He is believed to be a kind doctor Mark the letter A, B, C, or D on your answer sheet to indicate the word(s) OPPOSITE in meaning to the underlined word(s) in each of the following questions Question 10: Because Jack defaulted on his loan, the bank took him to court A failed to pay B paid in full C had a bad personality D was paid much money Question 11: My little daughter would spend an inordinate amount of time in the shop, deciding which goods she was going to buy A limited B required C abundant D excessive Mark the letter A, B, C, or D on your answer sheet to indicate the word(s) CLOSEST in meaning to the underlined word(s) in each of the following questions Question 12: I could see the finish line and thought I was home and dry A successful B hopeless C unsuccessful D hopeful Question 13: Dozens of applicants showed up for the vacant position, but only a handful of them were shortlisted for the interview A hand B class C small amount D small number Mark the letter A, B, C, or D on your answer sheet to indicate the underlined part that needs correction in each of the following questions Question 14: Nowhere you can enioy delicious dishes like here A like B Nowhere C dishes D you can Question 15: The room is so dirty that it needs to clean without delay A The B so C to clean D delay Question 16: The package must be wrapped careful before it is posted A must B is posted C The package D wrapped careful Read the following passage and mark the letter A, B, C, or D on your answer sheet to indicate the correct answer to each of the questions from 17 to 23 Before applying to a foreign university, one should consider the disadvantages of studying abroad First, a student may feel lonely by being far home and friends Also, difficulty in understanding a foreign language can be very disappointing and can affect the student’s grades It can be very expensive to pay the costs of travel and housing in a different country Finally, if there is an emergency at home, it will be hard to get home in a hurry However, studying in another country is advantageous in many ways A student is exposed to a new culture This culture teaches him or her about other people and other way of thinking, which can be promote friendships among countries He can learn an new language which may be beneficial for keeping up with research after student has finished studying He can learn experience not available in his country The student may also get the opportunity to study at a university where leading expert in his field may be teaching The leading expert can introduce the student to the most up-to-date finding of the tops researches in the field Exposure to such available knowledge and insights into the field can aid the student in becoming an expert as well Question 17: When studying abroad, a student A spends a lot of money for accommodation and transportation B All are correct C cannot go home immediately in emergency D is homesick and feel lonely Question 18: The word “available” in the second paragraph refers to A difficult B domestic C mischievous D ready Question 19: At a foreign university, a student may A meet a leading expert in his field B meet his neighbour C All are correct D travel a lot Question 20: When studying abroad, a student is exposed to a new culture that A causes some troubles B has no effects C has no advantage D enriches his knowledge about people and other ways of thinking Question 21: At a foreign university, a student A cannot learn a foreign language B doesn’t have to learn a language C can learn a foreign language D has to learn his mother tongue Question 22: The word “affect” in the first paragraph refers to A inform B invest C insist D impact Question 23: .to study abroad A There are neither advantages nor advantages B There are only disadvantages C There are only advantages D There are both advantages and disadvantages Read the following passage and mark the letter A, B, C, or D on your answer sheet to indicate the correct word or phrase that best fits each of the numbered blanks from 24 to 28 Water is necessary for life People can live only a few days (24) it Yet nearly 25 million people die each year because of it Both industrial nations and developing countries are worried about the (25) and quantity of water in the world Even though people, animals, agriculture, and industry use a lot of water, there is more than enough on the earth Water covers about three-quarters of the Earth’s surface However, 97.4 percent of it is salt water Three-fourths of the Earth’s fresh water is frozen in glaciers and in the great polar ice caps Most of the water we use (26) from rivers, lakes, and the atmosphere Less than one percent of the Earth’s water is usable, and we use it over and over again One of the (27) about water is distribution Water is not always distributed where the large (28) centres are Some regions get enough rain But it is all in one or two short rainy seasons Question 24:A for B without C with D in Question 25:A quality B situation C characteristics D conditions Question 26:A goes B gets C comes D arrives Question 27:A things B cases C problems D facts Question 28:A women B men C people D population Mark the letter A, B, C, or D on your answer sheet to indicate the sentence that best combines each pair of sentences in the following questions Question 29: I did not read his book I did not understand what the lecturer was saying A I found it very difficult to understand what the lecturer said in his book B What the lecturer wrote and said was too difficult for me to understand C I would have understood what the lecturer was saying ifI had read his book D The lecturer's book which I had not read was difficult to understand Question 30: They're my two sisters They aren't teachers like me A They're my two sisters, both of those are teachers like me B They're my two sisters who neither are teachers like me C They're my two sisters, neither of whom are teachers like me D Like me, neither of my two sisters aren't teachers Mark the letter A, B, C, or D on your answer sheet to indicate the correct answer to each of the following questions Question 31: The man said that he her the day before A will meet B had met C have met D met Question 32: Nam never comes to class on time and A neither does Hung B neither doesn’t Hung C so does Hung D so doesn’t Hung Question 33: It was in 1989 the Berlin Wall collapsed A which B that C what D when Question 34: .it with my own eyes, I would never have believed it A Had I not seen B Provided I had seen C Unless I had not seen D If I had seen Question 35: What are the .between women in old time and women in modem time? A difference B differs C differences D different Question 36: Sorry, I’m late, but my car on the way here, and I had to call the garage A not working B out of order C broke down D was broke Question 37: I .an old friend of mine in the street this morning We haven’t seen each other for ages A ran into B came round C ran out D came over Question 38: I remember .him a message yesterday A sending B to send C sends D sent Question 39: that he couldn’t go on working A He has so pain B Such was his pain C His pain was such D So was his pain Question 40: Charles was wearing at the party A a yellow silk funny tie B a funny wide yellow silk tie C very funny wide yellow silk tie D a tie yellow silk funny Question 41: Marie Curie was the first and only woman .two Nobel prizes A to be won B who win C to have won D that win Question 42: .you study for these exams .you will A The harder - the better B The more hard - the more good C The more - the much D The hardest - the best Read the following passage and mark, the letter A, B, C, or D on your answer sheet to indicate the correct answer to each of the questions from 43 to 50 After inventing dynamite, Swedish-bom Alfred Nobel became a very rich man However, he foresaw its universally destructive powers too late Nobel preferred not to be remembered as the inventor of dynamite, so in 1895, just two weeks before his death, he created a fund to be used for awarding prizes to people who had made worthwhile contributions to humanity Originally, there were five awards: literature, physics, chemistry, medicine, and peace Economics was added in 1968, just sixty-seven years after the first awards ceremony Nobel's original legacy of nine million dollars was invested, and the interest on this sum is used for the awards which vary from $30,000 to $125,000 Every year on December 10, the Einniversary of Nobel's death, the awsirds (gold medal, illuminated diploma, and money) are presented to the winners Sometimes politics plays an important role in the judges' decisions Americans have won numerous science awards, but relatively few literature prizes No awards were presented from 1940 to 1942 at the beginning of World War II Some people have won two prizes, but this is rare; others have shared their prizes Question 43: In the first paragraph, "worthwhile" is closest in meaning to A trivial B economic C prestigious D valuable Question 44: It is implied that Nobel's profession was in A economics B science C medicine D literature Question 45: What is the main idea of this passage? A Alfred Nobel left all of his money to science B Alfred Nobel made a lasting contribution to humanity C Alfred Nobel created awards in six categories for contributions to humanity D Alfred Nobel became very rich when he invented dynamite Question 46: The word "foresaw" in the first paragraph is nearest in meaning to A postponed B predicted C prevented D prevailed Question 47: In which area have Americans received the most awards? A Peace B Economics C Literature D Science Question 48: The Nobel prize was established in order to A spend money B resolve political differences C recognize worthwhile contributions to humanity D honour the inventor of dynamite Question 49: All of the following statements are true EXCEPT A Politics plays an important role in selecting the winners B A few individuals have won two awards C Awards vary in monetary value D Ceremonies are held on December 10 to commemorate Nobel's invention Question 50: The word "legacy" in the second paragraph means most nearly the same as A prize B legend C bequest D debt SỞ GIÁO DỤC ĐÀO TẠO ĐỀ CHÍNH THỨC (Đề gồm có 03 trang) ÔN THI TỐT NGHIỆP TRUNG HỌC PHỔ THÔNG NĂM HỌC 2016- 2017 MÔN TIẾNG ANH ~ ĐỀ 253 Thời gian: 60 phút - không tính thời gian giao đề Mark(s) Phách ……… Mark the letter A, B, C, or D on your answer sheet to indicate the word whose underlined part differs from the other three in pronunciation in each of the following questions Question 1:A measure B pressure C pleasure D leisure Question 2:A supported B completed C composed D defended Mark the letter A, B, C, or D on your answer sheet to indicate the underlined part that needs correction in each of the following questions Question 3: Nowhere you can enioy delicious dishes like here A dishes B you can C like D Nowhere Question 4: The room is so dirty that it needs to clean without delay A The B so C to clean D delay Question 5: The package must be wrapped careful before it is posted A The package B wrapped careful C must D is posted Read the following passage and mark the letter A, B, C, or D on your answer sheet to indicate the correct word or phrase that best fits each of the numbered blanks from 06 to 10 Water is necessary for life People can live only a few days (6) it Yet nearly 25 million people die each year because of it Both industrial nations and developing countries are worried about the (7) and quantity of water in the world Even though people, animals, agriculture, and industry use a lot of water, there is more than enough on the earth Water covers about three-quarters of the Earth’s surface However, 97.4 percent of it is salt water Three-fourths of the Earth’s fresh water is frozen in glaciers and in the great polar ice caps Most of the water we use (8) from rivers, lakes, and the atmosphere Less than one percent of the Earth’s water is usable, and we use it over and over again One of the (9) about water is distribution Water is not always distributed where the large (10) centres are Some regions get enough rain But it is all in one or two short rainy seasons Question 6:A for B without C in D with Question 7:A situation B quality C conditions D characteristics Question 8:A goes B comes C gets D arrives Question 9:A facts B problems C things D cases Question 10:A men B people C population D women Mark the letter A, B, C, or D on your answer sheet to indicate the correct answer to each of the following questions Question 11: Charles was wearing at the party A a tie yellow silk funny B very funny wide yellow silk tie C a funny wide yellow silk tie D a yellow silk funny tie Question 12: The man said that he her the day before A had met B will meet C have met D met Question 13: What are the .between women in old time and women in modem time? A differences B different C difference D differs Question 14: It was in 1989 the Berlin Wall collapsed A when B that C which D what Question 15: I remember .him a message yesterday A sent B sending C sends D to send Question 16: .it with my own eyes, I would never have believed it A If I had seen B Had I not seen C Provided I had seen D Unless I had not seen Question 17: .you study for these exams .you will A The more - the much B The more hard - the more good C The hardest - the best D The harder - the better Question 18: Nam never comes to class on time and A so doesn’t Hung B so does Hung C neither doesn’t Hung D neither does Hung Question 19: Sorry, I’m late, but my car on the way here, and I had to call the garage A out of order B not working C broke down D was broke Question 20: Marie Curie was the first and only woman .two Nobel prizes A to have won B to be won C that win D who win Question 21: that he couldn’t go on working A Such was his pain B He has so pain C His pain was such D So was his pain Question 22: I .an old friend of mine in the street this morning We haven’t seen each other for ages A ran out B ran into C came round D came over 10 Mark the letter A, B, C, or D on your answer sheet to indicate the most suitable response to complete each of the following exchanges Question 23: ~ Nam: "May I take my break now?" ~ Huy: " " A I think I need five minutes B No, you didn’t break it C I’ll be back in five minutes D Yes, of course Question 24: ~ Tim: " " ~ John: “Never mind.” A Sorry for staining your carpet Let me have it cleaned B Thank you for being honest with me C Congratulations! How wonderful! D Would you mind going to dinner next Sunday? Mark the letter A, B, C, or D on your answer sheet to indicate the word(s) CLOSEST in meaning to the underlined word(s) in each of the following questions Question 25: Dozens of applicants showed up for the vacant position, but only a handful of them were shortlisted for the interview A small number B small amount C hand D class Question 26: I could see the finish line and thought I was home and dry A hopeless B hopeful C unsuccessful D successful Mark the letter A, B, C, or D on your answer sheet to indicate the word that differs from the other three in the position of primary stress in each of the following questions Question 27:A wonderful B publisher C understand D character Question 28:A commitment B inflation C collection D enterprise Read the following passage and mark, the letter A, B, C, or D on your answer sheet to indicate the correct answer to each of the questions from 29 to 36 After inventing dynamite, Swedish-bom Alfred Nobel became a very rich man However, he foresaw its universally destructive powers too late Nobel preferred not to be remembered as the inventor of dynamite, so in 1895, just two weeks before his death, he created a fund to be used for awarding prizes to people who had made worthwhile contributions to humanity Originally, there were five awards: literature, physics, chemistry, medicine, and peace Economics was added in 1968, just sixty-seven years after the first awards ceremony Nobel's original legacy of nine million dollars was invested, and the interest on this sum is used for the awards which vary from $30,000 to $125,000 Every year on December 10, the Einniversary of Nobel's death, the awsirds (gold medal, illuminated diploma, and money) are presented to the winners Sometimes politics plays an important role in the judges' decisions Americans have won numerous science awards, but relatively few literature prizes No awards were presented from 1940 to 1942 at the beginning of World War II Some people have won two prizes, but this is rare; others have shared their prizes Question 29: The Nobel prize was established in order to A recognize worthwhile contributions to humanity B resolve political differences C honour the inventor of dynamite D spend money Question 30: In which area have Americans received the most awards? A Literature B Peace C Science D Economics Question 31: In the first paragraph, "worthwhile" is closest in meaning to A valuable B trivial C economic D prestigious Question 32: It is implied that Nobel's profession was in A literature B economics C medicine D science Question 33: All of the following statements are true EXCEPT A Ceremonies are held on December 10 to commemorate Nobel's invention B Politics plays an important role in selecting the winners C A few individuals have won two awards D Awards vary in monetary value Question 34: The word "legacy" in the second paragraph means most nearly the same as A bequest B prize C legend D debt Question 35: The word "foresaw" in the first paragraph is nearest in meaning to A prevented B postponed C prevailed D predicted Question 36: What is the main idea of this passage? A Alfred Nobel left all of his money to science B Alfred Nobel made a lasting contribution to humanity C Alfred Nobel became very rich when he invented dynamite D Alfred Nobel created awards in six categories for contributions to humanity Mark the letter A, B, C, or D on your answer sheet to indicate the sentence that best combines each pair of sentences in the following questions Question 37: They're my two sisters They aren't teachers like me A They're my two sisters, both of those are teachers like me B They're my two sisters, neither of whom are teachers like me C They're my two sisters who neither are teachers like me 11 D Like me, neither of my two sisters aren't teachers Question 38: I did not read his book I did not understand what the lecturer was saying A The lecturer's book which I had not read was difficult to understand B I would have understood what the lecturer was saying ifI had read his book C I found it very difficult to understand what the lecturer said in his book D What the lecturer wrote and said was too difficult for me to understand Mark the letter A, B, C, or D on your answer sheet to indicate the word(s) OPPOSITE in meaning to the underlined word(s) in each of the following questions Question 39: My little daughter would spend an inordinate amount of time in the shop, deciding which goods she was going to buy A abundant B limited C required D excessive Question 40: Because Jack defaulted on his loan, the bank took him to court A failed to pay B paid in full C was paid much money D had a bad personality Read the following passage and mark the letter A, B, C, or D on your answer sheet to indicate the correct answer to each of the questions from 41 to 47 Before applying to a foreign university, one should consider the disadvantages of studying abroad First, a student may feel lonely by being far home and friends Also, difficulty in understanding a foreign language can be very disappointing and can affect the student’s grades It can be very expensive to pay the costs of travel and housing in a different country Finally, if there is an emergency at home, it will be hard to get home in a hurry However, studying in another country is advantageous in many ways A student is exposed to a new culture This culture teaches him or her about other people and other way of thinking, which can be promote friendships among countries He can learn an new language which may be beneficial for keeping up with research after student has finished studying He can learn experience not available in his country The student may also get the opportunity to study at a university where leading expert in his field may be teaching The leading expert can introduce the student to the most up-to-date finding of the tops researches in the field Exposure to such available knowledge and insights into the field can aid the student in becoming an expert as well Question 41: .to study abroad A There are neither advantages nor advantages B There are only advantages C There are both advantages and disadvantages D There are only disadvantages Question 42: When studying abroad, a student is exposed to a new culture that A enriches his knowledge about people and other ways of thinking B causes some troubles C has no advantage D has no effects Question 43: The word “available” in the second paragraph refers to A ready B difficult C mischievous D domestic Question 44: The word “affect” in the first paragraph refers to A invest B inform C insist D impact Question 45: At a foreign university, a student A has to learn his mother tongue B doesn’t have to learn a language C can learn a foreign language D cannot learn a foreign language Question 46: When studying abroad, a student A All are correct B is homesick and feel lonely C spends a lot of money for accommodation and transportation D cannot go home immediately in emergency Question 47: At a foreign university, a student may A travel a lot B meet a leading expert in his field C All are correct D meet his neighbour Mark the letter A, B, C, or D on your answer sheet to indicate the sentence that is closest in meaning to each of the following questions Question 48: We lost the game because of my mistakes A Despite my mistakes, we didn’t lose the game B I made mistakes but we lost the game C Although I made mistakes, we didn’t lose the game D If I hadn ’t made mistakes, we wouldn’t have lost the game Question 49: People believe that he is a kind doctor A He is believed to be a kind doctor B He is really a kind doctor C Everybody admires him because he is a kind doctor D He is loved by people because he is a kind doctor Question 50: You can refuse to answer my question in no way A In no way can you refuse to answer my question B In no way you refuse to answer my question C In no way you can refuse to answer my question D In no way you refuse to answer my question 12 SỞ GIÁO DỤC ĐÀO TẠO ĐỀ CHÍNH THỨC (Đề gồm có 03 trang) ÔN THI TỐT NGHIỆP TRUNG HỌC PHỔ THÔNG NĂM HỌC 2016- 2017 MÔN TIẾNG ANH ~ ĐỀ 343 Thời gian: 60 phút - không tính thời gian giao đề Mark(s) Phách ……… Mark the letter A, B, C, or D on your answer sheet to indicate the word whose underlined part differs from the other three in pronunciation in each of the following questions Question 1:A supported B composed C defended D completed Question 2:A pressure B measure C pleasure D leisure Mark the letter A, B, C, or D on your answer sheet to indicate the sentence that is closest in meaning to each of the following questions Question 3: We lost the game because of my mistakes A Although I made mistakes, we didn’t lose the game B If I hadn ’t made mistakes, we wouldn’t have lost the game C Despite my mistakes, we didn’t lose the game D I made mistakes but we lost the game Question 4: People believe that he is a kind doctor A Everybody admires him because he is a kind doctor B He is believed to be a kind doctor C He is really a kind doctor D He is loved by people because he is a kind doctor Question 5: You can refuse to answer my question in no way A In no way you can refuse to answer my question B In no way you refuse to answer my question C In no way can you refuse to answer my question D In no way you refuse to answer my question Mark the letter A, B, C, or D on your answer sheet to indicate the underlined part that needs correction in each of the following questions Question 6: Nowhere you can enioy delicious dishes like here A dishes B you can C like D Nowhere Question 7: The package must be wrapped careful before it is posted A is posted B The package C wrapped careful D must Question 8: The room is so dirty that it needs to clean without delay A to clean B The C delay D so Mark the letter A, B, C, or D on your answer sheet to indicate the most suitable response to complete each of the following exchanges Question 9: ~ Nam: "May I take my break now?" ~ Huy: " " A No, you didn’t break it B Yes, of course C I think I need five minutes D I’ll be back in five minutes Question 10: ~ Tim: " " ~ John: “Never mind.” A Thank you for being honest with me B Sorry for staining your carpet Let me have it cleaned C Congratulations! How wonderful! D Would you mind going to dinner next Sunday? Mark the letter A, B, C, or D on your answer sheet to indicate the correct answer to each of the following questions Question 11: Sorry, I’m late, but my car on the way here, and I had to call the garage A not working B out of order C broke down D was broke Question 12: Marie Curie was the first and only woman .two Nobel prizes A who win B to have won C to be won D that win Question 13: The man said that he her the day before A will meet B had met C have met D met Question 14: I remember .him a message yesterday A sent B to send C sends D sending Question 15: .you study for these exams .you will A The hardest - the best B The more - the much C The harder - the better D The more hard - the more good Question 16: I .an old friend of mine in the street this morning We haven’t seen each other for ages A came round B came over C ran into D ran out Question 17: .it with my own eyes, I would never have believed it A Had I not seen B Unless I had not seen C Provided I had seen D If I had seen Question 18: It was in 1989 the Berlin Wall collapsed A that B which C what D when Question 19: Charles was wearing at the party 13 A a yellow silk funny tie B very funny wide yellow silk tie C a tie yellow silk funny D a funny wide yellow silk tie Question 20: Nam never comes to class on time and A neither does Hung B so doesn’t Hung C so does Hung D neither doesn’t Hung Question 21: that he couldn’t go on working A He has so pain B Such was his pain C His pain was such D So was his pain Question 22: What are the .between women in old time and women in modem time? A different B differs C difference D differences Mark the letter A, B, C, or D on your answer sheet to indicate the word(s) CLOSEST in meaning to the underlined word(s) in each of the following questions Question 23: Dozens of applicants showed up for the vacant position, but only a handful of them were shortlisted for the interview A small number B hand C class D small amount Question 24: I could see the finish line and thought I was home and dry A successful B hopeless C hopeful D unsuccessful Read the following passage and mark, the letter A, B, C, or D on your answer sheet to indicate the correct answer to each of the questions from 25 to 32 After inventing dynamite, Swedish-bom Alfred Nobel became a very rich man However, he foresaw its universally destructive powers too late Nobel preferred not to be remembered as the inventor of dynamite, so in 1895, just two weeks before his death, he created a fund to be used for awarding prizes to people who had made worthwhile contributions to humanity Originally, there were five awards: literature, physics, chemistry, medicine, and peace Economics was added in 1968, just sixty-seven years after the first awards ceremony Nobel's original legacy of nine million dollars was invested, and the interest on this sum is used for the awards which vary from $30,000 to $125,000 Every year on December 10, the Einniversary of Nobel's death, the awsirds (gold medal, illuminated diploma, and money) are presented to the winners Sometimes politics plays an important role in the judges' decisions Americans have won numerous science awards, but relatively few literature prizes No awards were presented from 1940 to 1942 at the beginning of World War II Some people have won two prizes, but this is rare; others have shared their prizes Question 25: It is implied that Nobel's profession was in A economics B science C literature D medicine Question 26: The word "foresaw" in the first paragraph is nearest in meaning to A postponed B prevailed C predicted D prevented Question 27: In the first paragraph, "worthwhile" is closest in meaning to A valuable B trivial C economic D prestigious Question 28: The Nobel prize was established in order to A honour the inventor of dynamite B resolve political differences C recognize worthwhile contributions to humanity D spend money Question 29: All of the following statements are true EXCEPT A Ceremonies are held on December 10 to commemorate Nobel's invention B A few individuals have won two awards C Awards vary in monetary value D Politics plays an important role in selecting the winners Question 30: In which area have Americans received the most awards? A Economics B Literature C Peace D Science Question 31: What is the main idea of this passage? A Alfred Nobel made a lasting contribution to humanity B Alfred Nobel created awards in six categories for contributions to humanity C Alfred Nobel left all of his money to science D Alfred Nobel became very rich when he invented dynamite Question 32: The word "legacy" in the second paragraph means most nearly the same as A legend B bequest C debt D prize Mark the letter A, B, C, or D on your answer sheet to indicate the word that differs from the other three in the position of primary stress in each of the following questions Question 33:A commitment B collection C enterprise D inflation Question 34:A understand B character C publisher D wonderful Read the following passage and mark the letter A, B, C, or D on your answer sheet to indicate the correct answer to each of the questions from 35 to 41 Before applying to a foreign university, one should consider the disadvantages of studying abroad First, a student may feel lonely by being far home and friends Also, difficulty in understanding a foreign language can be very disappointing and can affect the student’s grades It can be very expensive to pay the costs of travel and housing in a different country Finally, if there is an emergency at home, it will be hard to get home in a hurry 14 However, studying in another country is advantageous in many ways A student is exposed to a new culture This culture teaches him or her about other people and other way of thinking, which can be promote friendships among countries He can learn an new language which may be beneficial for keeping up with research after student has finished studying He can learn experience not available in his country The student may also get the opportunity to study at a university where leading expert in his field may be teaching The leading expert can introduce the student to the most up-to-date finding of the tops researches in the field Exposure to such available knowledge and insights into the field can aid the student in becoming an expert as well Question 35: The word “affect” in the first paragraph refers to A impact B insist C inform D invest Question 36: At a foreign university, a student A can learn a foreign language B cannot learn a foreign language C doesn’t have to learn a language D has to learn his mother tongue Question 37: At a foreign university, a student may A travel a lot B All are correct C meet a leading expert in his field D meet his neighbour Question 38: When studying abroad, a student A is homesick and feel lonely B cannot go home immediately in emergency C All are correct D spends a lot of money for accommodation and transportation Question 39: When studying abroad, a student is exposed to a new culture that A has no advantage B has no effects C enriches his knowledge about people and other ways of thinking D causes some troubles Question 40: .to study abroad A There are neither advantages nor advantages B There are both advantages and disadvantages C There are only disadvantages D There are only advantages Question 41: The word “available” in the second paragraph refers to A ready B difficult C mischievous D domestic Read the following passage and mark the letter A, B, C, or D on your answer sheet to indicate the correct word or phrase that best fits each of the numbered blanks from 42 to 46 Water is necessary for life People can live only a few days (42) it Yet nearly 25 million people die each year because of it Both industrial nations and developing countries are worried about the (43) and quantity of water in the world Even though people, animals, agriculture, and industry use a lot of water, there is more than enough on the earth Water covers about three-quarters of the Earth’s surface However, 97.4 percent of it is salt water Three-fourths of the Earth’s fresh water is frozen in glaciers and in the great polar ice caps Most of the water we use (44) from rivers, lakes, and the atmosphere Less than one percent of the Earth’s water is usable, and we use it over and over again One of the (45) about water is distribution Water is not always distributed where the large (46) centres are Some regions get enough rain But it is all in one or two short rainy seasons Question 42:A with B in C without D for Question 43:A quality B characteristics C conditions D situation Question 44:A comes B arrives C gets D goes Question 45:A cases B things C problems D facts Question 46:A women B people C population D men Mark the letter A, B, C, or D on your answer sheet to indicate the sentence that best combines each pair of sentences in the following questions Question 47: I did not read his book I did not understand what the lecturer was saying A The lecturer's book which I had not read was difficult to understand B I found it very difficult to understand what the lecturer said in his book C I would have understood what the lecturer was saying ifI had read his book D What the lecturer wrote and said was too difficult for me to understand Question 48: They're my two sisters They aren't teachers like me A They're my two sisters who neither are teachers like me B They're my two sisters, both of those are teachers like me C Like me, neither of my two sisters aren't teachers D They're my two sisters, neither of whom are teachers like me Mark the letter A, B, C, or D on your answer sheet to indicate the word(s) OPPOSITE in meaning to the underlined word(s) in each of the following questions Question 49: My little daughter would spend an inordinate amount of time in the shop, deciding which goods she was going to buy A limited B excessive C required D abundant Question 50: Because Jack defaulted on his loan, the bank took him to court A was paid much money B paid in full C failed to pay D had a bad personality 15 SỞ GIÁO DỤC ĐÀO TẠO ĐỀ CHÍNH THỨC (Đề gồm có 03 trang) ÔN THI TỐT NGHIỆP TRUNG HỌC PHỔ THÔNG NĂM HỌC 2016- 2017 MÔN TIẾNG ANH ~ ĐỀ 659 Thời gian: 60 phút - không tính thời gian giao đề Mark(s) Phách ……… Mark the letter A, B, C, or D on your answer sheet to indicate the sentence that best combines each pair of sentences in the following questions Question 1: They're my two sisters They aren't teachers like me A They're my two sisters who neither are teachers like me B They're my two sisters, neither of whom are teachers like me C Like me, neither of my two sisters aren't teachers D They're my two sisters, both of those are teachers like me Question 2: I did not read his book I did not understand what the lecturer was saying A I found it very difficult to understand what the lecturer said in his book B I would have understood what the lecturer was saying ifI had read his book C The lecturer's book which I had not read was difficult to understand D What the lecturer wrote and said was too difficult for me to understand Mark the letter A, B, C, or D on your answer sheet to indicate the underlined part that needs correction in each of the following questions Question 3: The room is so dirty that it needs to clean without delay A delay B to clean C so D The Question 4: Nowhere you can enioy delicious dishes like here A like B you can C Nowhere D dishes Question 5: The package must be wrapped careful before it is posted A The package B must C wrapped careful D is posted Read the following passage and mark the letter A, B, C, or D on your answer sheet to indicate the correct word or phrase that best fits each of the numbered blanks from 06 to 10 Water is necessary for life People can live only a few days (6) it Yet nearly 25 million people die each year because of it Both industrial nations and developing countries are worried about the (7) and quantity of water in the world Even though people, animals, agriculture, and industry use a lot of water, there is more than enough on the earth Water covers about three-quarters of the Earth’s surface However, 97.4 percent of it is salt water Three-fourths of the Earth’s fresh water is frozen in glaciers and in the great polar ice caps Most of the water we use (8) from rivers, lakes, and the atmosphere Less than one percent of the Earth’s water is usable, and we use it over and over again One of the (9) about water is distribution Water is not always distributed where the large (10) centres are Some regions get enough rain But it is all in one or two short rainy seasons Question 6:A without B in C for D with Question 7:A situation B conditions C quality D characteristics Question 8:A goes B comes C arrives D gets Question 9:A facts B things C cases D problems Question 10:A men B population C women D people Mark the letter A, B, C, or D on your answer sheet to indicate the correct answer to each of the following questions Question 11: It was in 1989 the Berlin Wall collapsed A what B that C which D when Question 12: .you study for these exams .you will A The more hard - the more good B The more - the much C The hardest - the best D The harder - the better Question 13: Charles was wearing at the party A a yellow silk funny tie B a tie yellow silk funny C very funny wide yellow silk tie D a funny wide yellow silk tie Question 14: Sorry, I’m late, but my car on the way here, and I had to call the garage A not working B out of order C was broke D broke down Question 15: The man said that he her the day before A had met B have met C met D will meet Question 16: What are the .between women in old time and women in modem time? A different B differences C differs D difference Question 17: .it with my own eyes, I would never have believed it A Provided I had seen B Unless I had not seen C If I had seen D Had I not seen Question 18: I .an old friend of mine in the street this morning We haven’t seen each other for ages A came round B ran into C ran out D came over 16 Question 19: Nam never comes to class on time and A so doesn’t Hung B neither doesn’t Hung C so does Hung D neither does Hung Question 20: Marie Curie was the first and only woman .two Nobel prizes A that win B to be won C to have won D who win Question 21: that he couldn’t go on working A He has so pain B So was his pain C Such was his pain D His pain was such Question 22: I remember .him a message yesterday A to send B sends C sent D sending Mark the letter A, B, C, or D on your answer sheet to indicate the word(s) OPPOSITE in meaning to the underlined word(s) in each of the following questions Question 23: My little daughter would spend an inordinate amount of time in the shop, deciding which goods she was going to buy A abundant B required C limited D excessive Question 24: Because Jack defaulted on his loan, the bank took him to court A failed to pay B was paid much money C paid in full D had a bad personality Mark the letter A, B, C, or D on your answer sheet to indicate the word(s) CLOSEST in meaning to the underlined word(s) in each of the following questions Question 25: Dozens of applicants showed up for the vacant position, but only a handful of them were shortlisted for the interview A class B hand C small number D small amount Question 26: I could see the finish line and thought I was home and dry A hopeful B hopeless C successful D unsuccessful Mark the letter A, B, C, or D on your answer sheet to indicate the most suitable response to complete each of the following exchanges Question 27: ~ Nam: "May I take my break now?" ~ Huy: " " A Yes, of course B No, you didn’t break it C I’ll be back in five minutes D I think I need five minutes Question 28: ~ Tim: " " ~ John: “Never mind.” A Thank you for being honest with me B Sorry for staining your carpet Let me have it cleaned C Would you mind going to dinner next Sunday? D Congratulations! How wonderful! Mark the letter A, B, C, or D on your answer sheet to indicate the word whose underlined part differs from the other three in pronunciation in each of the following questions Question 29:A supported B defended C composed D completed Question 30:A measure B pressure C leisure D pleasure Mark the letter A, B, C, or D on your answer sheet to indicate the word that differs from the other three in the position of primary stress in each of the following questions Question 31:A publisher B understand C wonderful D character Question 32:A collection B inflation C commitment D enterprise Mark the letter A, B, C, or D on your answer sheet to indicate the sentence that is closest in meaning to each of the following questions Question 33: You can refuse to answer my question in no way A In no way you refuse to answer my question B In no way can you refuse to answer my question C In no way you can refuse to answer my question D In no way you refuse to answer my question Question 34: People believe that he is a kind doctor A He is believed to be a kind doctor B He is loved by people because he is a kind doctor C He is really a kind doctor D Everybody admires him because he is a kind doctor Question 35: We lost the game because of my mistakes A I made mistakes but we lost the game B Although I made mistakes, we didn’t lose the game C If I hadn ’t made mistakes, we wouldn’t have lost the game D Despite my mistakes, we didn’t lose the game Read the following passage and mark the letter A, B, C, or D on your answer sheet to indicate the correct answer to each of the questions from 36 to 42 Before applying to a foreign university, one should consider the disadvantages of studying abroad First, a student may feel lonely by being far home and friends Also, difficulty in understanding a foreign language can be very disappointing and can affect the student’s grades It can be very expensive to pay the costs of travel and housing in a different country Finally, if there is an emergency at home, it will be hard to get home in a hurry However, studying in another country is advantageous in many ways A student is exposed to a new culture This culture teaches him or her about other people and other way of thinking, which can be promote friendships among countries He can learn an new language which may be beneficial for keeping up with research after student has finished studying He can learn 17 experience not available in his country The student may also get the opportunity to study at a university where leading expert in his field may be teaching The leading expert can introduce the student to the most up-to-date finding of the tops researches in the field Exposure to such available knowledge and insights into the field can aid the student in becoming an expert as well Question 36: At a foreign university, a student may A meet a leading expert in his field B travel a lot C meet his neighbour D All are correct Question 37: When studying abroad, a student is exposed to a new culture that A has no advantage B has no effects C causes some troubles D enriches his knowledge about people and other ways of thinking Question 38: The word “affect” in the first paragraph refers to A invest B impact C inform D insist Question 39: At a foreign university, a student A can learn a foreign language B cannot learn a foreign language C has to learn his mother tongue D doesn’t have to learn a language Question 40: .to study abroad A There are only disadvantages B There are neither advantages nor advantages C There are both advantages and disadvantages D There are only advantages Question 41: When studying abroad, a student A cannot go home immediately in emergency B is homesick and feel lonely C All are correct D spends a lot of money for accommodation and transportation Question 42: The word “available” in the second paragraph refers to A difficult B mischievous C domestic D ready Read the following passage and mark, the letter A, B, C, or D on your answer sheet to indicate the correct answer to each of the questions from 43 to 50 After inventing dynamite, Swedish-bom Alfred Nobel became a very rich man However, he foresaw its universally destructive powers too late Nobel preferred not to be remembered as the inventor of dynamite, so in 1895, just two weeks before his death, he created a fund to be used for awarding prizes to people who had made worthwhile contributions to humanity Originally, there were five awards: literature, physics, chemistry, medicine, and peace Economics was added in 1968, just sixty-seven years after the first awards ceremony Nobel's original legacy of nine million dollars was invested, and the interest on this sum is used for the awards which vary from $30,000 to $125,000 Every year on December 10, the Einniversary of Nobel's death, the awsirds (gold medal, illuminated diploma, and money) are presented to the winners Sometimes politics plays an important role in the judges' decisions Americans have won numerous science awards, but relatively few literature prizes No awards were presented from 1940 to 1942 at the beginning of World War II Some people have won two prizes, but this is rare; others have shared their prizes Question 43: The word "foresaw" in the first paragraph is nearest in meaning to A postponed B prevailed C prevented D predicted Question 44: All of the following statements are true EXCEPT A A few individuals have won two awards B Politics plays an important role in selecting the winners C Ceremonies are held on December 10 to commemorate Nobel's invention D Awards vary in monetary value Question 45: What is the main idea of this passage? A Alfred Nobel left all of his money to science B Alfred Nobel became very rich when he invented dynamite C Alfred Nobel created awards in six categories for contributions to humanity D Alfred Nobel made a lasting contribution to humanity Question 46: In which area have Americans received the most awards? A Economics B Science C Literature D Peace Question 47: The Nobel prize was established in order to A recognize worthwhile contributions to humanity B resolve political differences C honour the inventor of dynamite D spend money Question 48: In the first paragraph, "worthwhile" is closest in meaning to A prestigious B economic C valuable D trivial Question 49: It is implied that Nobel's profession was in A literature B economics C science D medicine Question 50: The word "legacy" in the second paragraph means most nearly the same as A prize B legend C debt D bequest 18 SỞ GIÁO DỤC ĐÀO TẠO ĐỀ CHÍNH THỨC (Đề gồm có 03 trang) ÔN THI TỐT NGHIỆP TRUNG HỌC PHỔ THÔNG NĂM HỌC 2016- 2017 MÔN TIẾNG ANH ~ ĐỀ 288 Thời gian: 60 phút - không tính thời gian giao đề Mark(s) Phách ……… Mark the letter A, B, C, or D on your answer sheet to indicate the sentence that is closest in meaning to each of the following questions Question 1: We lost the game because of my mistakes A Despite my mistakes, we didn’t lose the game B If I hadn ’t made mistakes, we wouldn’t have lost the game C I made mistakes but we lost the game D Although I made mistakes, we didn’t lose the game Question 2: People believe that he is a kind doctor A Everybody admires him because he is a kind doctor B He is believed to be a kind doctor C He is really a kind doctor D He is loved by people because he is a kind doctor Question 3: You can refuse to answer my question in no way A In no way can you refuse to answer my question B In no way you refuse to answer my question C In no way you can refuse to answer my question D In no way you refuse to answer my question Mark the letter A, B, C, or D on your answer sheet to indicate the most suitable response to complete each of the following exchanges Question 4: ~ Tim: " " ~ John: “Never mind.” A Sorry for staining your carpet Let me have it cleaned B Would you mind going to dinner next Sunday? C Congratulations! How wonderful! D Thank you for being honest with me Question 5: ~ Nam: "May I take my break now?" ~ Huy: " " A I think I need five minutes B Yes, of course C No, you didn’t break it D I’ll be back in five minutes Mark the letter A, B, C, or D on your answer sheet to indicate the underlined part that needs correction in each of the following questions Question 6: Nowhere you can enioy delicious dishes like here A Nowhere B like C dishes D you can Question 7: The package must be wrapped careful before it is posted A is posted B The package C wrapped careful D must Question 8: The room is so dirty that it needs to clean without delay A The B delay C so D to clean Mark the letter A, B, C, or D on your answer sheet to indicate the word whose underlined part differs from the other three in pronunciation in each of the following questions Question 9:A measure B pressure C leisure D pleasure Question 10:A defended B composed C completed D supported Mark the letter A, B, C, or D on your answer sheet to indicate the word that differs from the other three in the position of primary stress in each of the following questions Question 11:A character B wonderful C publisher D understand Question 12:A collectionB commitment C enterprise D inflation Mark the letter A, B, C, or D on your answer sheet to indicate the word(s) CLOSEST in meaning to the underlined word(s) in each of the following questions Question 13: Dozens of applicants showed up for the vacant position, but only a handful of them were shortlisted for the interview A class B small number C small amount D hand Question 14: I could see the finish line and thought I was home and dry A successful B unsuccessful C hopeful D hopeless Mark the letter A, B, C, or D on your answer sheet to indicate the correct answer to each of the following questions Question 15: Charles was wearing at the party A a funny wide yellow silk tie B a yellow silk funny tie C very funny wide yellow silk tie D a tie yellow silk funny Question 16: Marie Curie was the first and only woman .two Nobel prizes A to be won B that win C who win D to have won Question 17: .you study for these exams .you will A The hardest - the best B The more - the much 19 C The harder - the better D The more hard - the more good Question 18: Sorry, I’m late, but my car on the way here, and I had to call the garage A broke down B not working C out of order D was broke Question 19: that he couldn’t go on working A He has so pain B His pain was such C So was his pain D Such was his pain Question 20: .it with my own eyes, I would never have believed it A Had I not seen B If I had seen C Provided I had seen D Unless I had not seen Question 21: The man said that he her the day before A have met B met C will meet D had met Question 22: I .an old friend of mine in the street this morning We haven’t seen each other for ages A ran out B came round C ran into D came over Question 23: It was in 1989 the Berlin Wall collapsed A what B which C when D that Question 24: I remember .him a message yesterday A to send B sent C sends D sending Question 25: Nam never comes to class on time and A so doesn’t Hung B neither does Hung C so does Hung D neither doesn’t Hung Question 26: What are the .between women in old time and women in modem time? A differences B differs C different D difference Mark the letter A, B, C, or D on your answer sheet to indicate the word(s) OPPOSITE in meaning to the underlined word(s) in each of the following questions Question 27: Because Jack defaulted on his loan, the bank took him to court A was paid much money B had a bad personality C failed to pay D paid in full Question 28: My little daughter would spend an inordinate amount of time in the shop, deciding which goods she was going to buy A limited B abundant C excessive D required Mark the letter A, B, C, or D on your answer sheet to indicate the sentence that best combines each pair of sentences in the following questions Question 29: I did not read his book I did not understand what the lecturer was saying A I would have understood what the lecturer was saying ifI had read his book B I found it very difficult to understand what the lecturer said in his book C What the lecturer wrote and said was too difficult for me to understand D The lecturer's book which I had not read was difficult to understand Question 30: They're my two sisters They aren't teachers like me A They're my two sisters who neither are teachers like me B They're my two sisters, both of those are teachers like me C They're my two sisters, neither of whom are teachers like me D Like me, neither of my two sisters aren't teachers Read the following passage and mark the letter A, B, C, or D on your answer sheet to indicate the correct word or phrase that best fits each of the numbered blanks from 31 to 35 Water is necessary for life People can live only a few days (31) it Yet nearly 25 million people die each year because of it Both industrial nations and developing countries are worried about the (32) and quantity of water in the world Even though people, animals, agriculture, and industry use a lot of water, there is more than enough on the earth Water covers about three-quarters of the Earth’s surface However, 97.4 percent of it is salt water Three-fourths of the Earth’s fresh water is frozen in glaciers and in the great polar ice caps Most of the water we use (33) from rivers, lakes, and the atmosphere Less than one percent of the Earth’s water is usable, and we use it over and over again One of the (34) about water is distribution Water is not always distributed where the large (35) centres are Some regions get enough rain But it is all in one or two short rainy seasons Question 31:A in B without C for D with Question 32:A characteristics B conditions C situation D quality Question 33:A comes B goes C gets D arrives Question 34:A facts B things C problems D cases Question 35:A women B population C people D men Read the following passage and mark the letter A, B, C, or D on your answer sheet to indicate the correct answer to each of the questions from 36 to 42 Before applying to a foreign university, one should consider the disadvantages of studying abroad First, a student may feel lonely by being far home and friends Also, difficulty in understanding a foreign language can be very disappointing and can affect the student’s grades It can be very expensive to pay the costs of travel and housing in a different country Finally, if there is an emergency at home, it will be hard to get home in a hurry However, studying in another country is advantageous in many ways A student is exposed to a new culture This culture teaches him or her about other people and other way of thinking, which can be promote friendships among countries He can learn an new language which may be beneficial for keeping up with research after student has finished studying He can learn 20 experience not available in his country The student may also get the opportunity to study at a university where leading expert in his field may be teaching The leading expert can introduce the student to the most up-to-date finding of the tops researches in the field Exposure to such available knowledge and insights into the field can aid the student in becoming an expert as well Question 36: The word “affect” in the first paragraph refers to A invest B insist C inform D impact Question 37: When studying abroad, a student is exposed to a new culture that A has no advantage B causes some troubles C enriches his knowledge about people and other ways of thinking D has no effects Question 38: When studying abroad, a student A is homesick and feel lonely B All are correct C cannot go home immediately in emergency D spends a lot of money for accommodation and transportation Question 39: The word “available” in the second paragraph refers to A difficult B mischievous C domestic D ready Question 40: At a foreign university, a student may A travel a lot B meet a leading expert in his field C meet his neighbour D All are correct Question 41: At a foreign university, a student A can learn a foreign language B doesn’t have to learn a language C has to learn his mother tongue D cannot learn a foreign language Question 42: .to study abroad A There are both advantages and disadvantages B There are neither advantages nor advantages C There are only advantages D There are only disadvantages Read the following passage and mark, the letter A, B, C, or D on your answer sheet to indicate the correct answer to each of the questions from 43 to 50 After inventing dynamite, Swedish-bom Alfred Nobel became a very rich man However, he foresaw its universally destructive powers too late Nobel preferred not to be remembered as the inventor of dynamite, so in 1895, just two weeks before his death, he created a fund to be used for awarding prizes to people who had made worthwhile contributions to humanity Originally, there were five awards: literature, physics, chemistry, medicine, and peace Economics was added in 1968, just sixty-seven years after the first awards ceremony Nobel's original legacy of nine million dollars was invested, and the interest on this sum is used for the awards which vary from $30,000 to $125,000 Every year on December 10, the Einniversary of Nobel's death, the awsirds (gold medal, illuminated diploma, and money) are presented to the winners Sometimes politics plays an important role in the judges' decisions Americans have won numerous science awards, but relatively few literature prizes No awards were presented from 1940 to 1942 at the beginning of World War II Some people have won two prizes, but this is rare; others have shared their prizes Question 43: The Nobel prize was established in order to A spend money B recognize worthwhile contributions to humanity C honour the inventor of dynamite D resolve political differences Question 44: In which area have Americans received the most awards? A Literature B Peace C Science D Economics Question 45: The word "foresaw" in the first paragraph is nearest in meaning to A postponed B prevented C prevailed D predicted Question 46: What is the main idea of this passage? A Alfred Nobel created awards in six categories for contributions to humanity B Alfred Nobel left all of his money to science C Alfred Nobel became very rich when he invented dynamite D Alfred Nobel made a lasting contribution to humanity Question 47: All of the following statements are true EXCEPT A Politics plays an important role in selecting the winners B Awards vary in monetary value C A few individuals have won two awards D Ceremonies are held on December 10 to commemorate Nobel's invention Question 48: It is implied that Nobel's profession was in A medicine B literature C science D economics Question 49: The word "legacy" in the second paragraph means most nearly the same as A debt B legend C bequest D prize Question 50: In the first paragraph, "worthwhile" is closest in meaning to A trivial B economic C prestigious D valuable 21 SỞ GIÁO DỤC ĐÀO TẠO ĐỀ CHÍNH THỨC (Đề gồm có 03 trang) ÔN THI TỐT NGHIỆP TRUNG HỌC PHỔ THÔNG NĂM HỌC 2016- 2017 MÔN TIẾNG ANH ~ ĐỀ 857 Thời gian: 60 phút - không tính thời gian giao đề Mark(s) Phách ……… Mark the letter A, B, C, or D on your answer sheet to indicate the underlined part that needs correction in each of the following questions Question 1: The room is so dirty that it needs to clean without delay A The B so C to clean D delay Question 2: Nowhere you can enioy delicious dishes like here A Nowhere B you can C like D dishes Question 3: The package must be wrapped careful before it is posted A wrapped careful B is posted C The package D must Mark the letter A, B, C, or D on your answer sheet to indicate the word that differs from the other three in the position of primary stress in each of the following questions Question 4:A inflation B commitment C enterprise D collection Question 5:A understandB character C wonderful D publisher Mark the letter A, B, C, or D on your answer sheet to indicate the word(s) OPPOSITE in meaning to the underlined word(s) in each of the following questions Question 6: Because Jack defaulted on his loan, the bank took him to court A paid in full B was paid much money C failed to pay D had a bad personality Question 7: My little daughter would spend an inordinate amount of time in the shop, deciding which goods she was going to buy A excessive B required C abundant D limited Read the following passage and mark the letter A, B, C, or D on your answer sheet to indicate the correct word or phrase that best fits each of the numbered blanks from 08 to 12 Water is necessary for life People can live only a few days (8) it Yet nearly 25 million people die each year because of it Both industrial nations and developing countries are worried about the (9) and quantity of water in the world Even though people, animals, agriculture, and industry use a lot of water, there is more than enough on the earth Water covers about three-quarters of the Earth’s surface However, 97.4 percent of it is salt water Three-fourths of the Earth’s fresh water is frozen in glaciers and in the great polar ice caps Most of the water we use (10) from rivers, lakes, and the atmosphere Less than one percent of the Earth’s water is usable, and we use it over and over again One of the (11) about water is distribution Water is not always distributed where the large (12) centres are Some regions get enough rain But it is all in one or two short rainy seasons Question 8:A with B for C without D in Question 9:A situation B quality C conditions D characteristics Question 10:A gets B comes C goes D arrives Question 11:A cases B problems C things D facts Question 12:A women B men C people D population Mark the letter A, B, C, or D on your answer sheet to indicate the most suitable response to complete each of the following exchanges Question 13: ~ Nam: "May I take my break now?" ~ Huy: " " A No, you didn’t break it B I think I need five minutes C I’ll be back in five minutes D Yes, of course Question 14: ~ Tim: " " ~ John: “Never mind.” A Would you mind going to dinner next Sunday? B Thank you for being honest with me C Congratulations! How wonderful! D Sorry for staining your carpet Let me have it cleaned Mark the letter A, B, C, or D on your answer sheet to indicate the word(s) CLOSEST in meaning to the underlined word(s) in each of the following questions Question 15: Dozens of applicants showed up for the vacant position, but only a handful of them were shortlisted for the interview A class B small number C hand D small amount Question 16: I could see the finish line and thought I was home and dry A hopeless B unsuccessful C successful D hopeful Mark the letter A, B, C, or D on your answer sheet to indicate the word whose underlined part differs from the other three in pronunciation in each of the following questions Question 17:A pleasure B leisure C pressure D measure Question 18:A supported B completed C defended D composed 22  Mark the letter A, B, C, or D on your answer sheet to indicate the sentence that is closest in meaning to each of the following questions Question 19: People believe that he is a kind doctor A He is believed to be a kind doctor B Everybody admires him because he is a kind doctor C He is really a kind doctor D He is loved by people because he is a kind doctor Question 20: You can refuse to answer my question in no way A In no way you can refuse to answer my question B In no way can you refuse to answer my question C In no way you refuse to answer my question D In no way you refuse to answer my question Question 21: We lost the game because of my mistakes A Although I made mistakes, we didn’t lose the game B Despite my mistakes, we didn’t lose the game C If I hadn ’t made mistakes, we wouldn’t have lost the game D I made mistakes but we lost the game Mark the letter A, B, C, or D on your answer sheet to indicate the sentence that best combines each pair of sentences in the following questions Question 22: They're my two sisters They aren't teachers like me A Like me, neither of my two sisters aren't teachers B They're my two sisters, both of those are teachers like me C They're my two sisters, neither of whom are teachers like me D They're my two sisters who neither are teachers like me Question 23: I did not read his book I did not understand what the lecturer was saying A I would have understood what the lecturer was saying ifI had read his book B What the lecturer wrote and said was too difficult for me to understand C The lecturer's book which I had not read was difficult to understand D I found it very difficult to understand what the lecturer said in his book Mark the letter A, B, C, or D on your answer sheet to indicate the correct answer to each of the following questions Question 24: The man said that he her the day before A have met B will meet C met D had met Question 25: What are the .between women in old time and women in modem time? A differs B different C difference D differences Question 26: .you study for these exams .you will A The hardest - the best B The harder - the better C The more hard - the more good D The more - the much Question 27: It was in 1989 the Berlin Wall collapsed A that B what C which D when Question 28: that he couldn’t go on working A His pain was such B He has so pain C So was his pain D Such was his pain Question 29: Charles was wearing at the party A a tie yellow silk funny B very funny wide yellow silk tie C a funny wide yellow silk tie D a yellow silk funny tie Question 30: Nam never comes to class on time and A so does Hung B so doesn’t Hung C neither does Hung D neither doesn’t Hung Question 31: .it with my own eyes, I would never have believed it A Unless I had not seen B Had I not seen C If I had seen D Provided I had seen Question 32: Sorry, I’m late, but my car on the way here, and I had to call the garage A not working B broke down C out of order D was broke Question 33: Marie Curie was the first and only woman .two Nobel prizes A who win B to have won C to be won D that win Question 34: I .an old friend of mine in the street this morning We haven’t seen each other for ages A ran into B ran out C came round D came over Question 35: I remember .him a message yesterday A to send B sends C sending D sent Read the following passage and mark the letter A, B, C, or D on your answer sheet to indicate the correct answer to each of the questions from 36 to 42 Before applying to a foreign university, one should consider the disadvantages of studying abroad First, a student may feel lonely by being far home and friends Also, difficulty in understanding a foreign language can be very disappointing and can affect the student’s grades It can be very expensive to pay the costs of travel and housing in a different country Finally, if there is an emergency at home, it will be hard to get home in a hurry However, studying in another country is advantageous in many ways A student is exposed to a new culture This culture teaches him or her about other people and other way of thinking, which can be promote friendships among countries He can 23 learn an new language which may be beneficial for keeping up with research after student has finished studying He can learn experience not available in his country The student may also get the opportunity to study at a university where leading expert in his field may be teaching The leading expert can introduce the student to the most up-to-date finding of the tops researches in the field Exposure to such available knowledge and insights into the field can aid the student in becoming an expert as well Question 36: The word “affect” in the first paragraph refers to A invest B impact C insist D inform Question 37: At a foreign university, a student A doesn’t have to learn a language B can learn a foreign language C has to learn his mother tongue D cannot learn a foreign language Question 38: At a foreign university, a student may A All are correct B travel a lot C meet his neighbour D meet a leading expert in his field Question 39: When studying abroad, a student is exposed to a new culture that A has no effects B has no advantage C enriches his knowledge about people and other ways of thinking D causes some troubles Question 40: When studying abroad, a student A is homesick and feel lonely B cannot go home immediately in emergency C spends a lot of money for accommodation and transportation D All are correct Question 41: .to study abroad A There are only disadvantages B There are only advantages C There are neither advantages nor advantages D There are both advantages and disadvantages Question 42: The word “available” in the second paragraph refers to A difficult B ready C domestic D mischievous Read the following passage and mark, the letter A, B, C, or D on your answer sheet to indicate the correct answer to each of the questions from 43 to 50 After inventing dynamite, Swedish-bom Alfred Nobel became a very rich man However, he foresaw its universally destructive powers too late Nobel preferred not to be remembered as the inventor of dynamite, so in 1895, just two weeks before his death, he created a fund to be used for awarding prizes to people who had made worthwhile contributions to humanity Originally, there were five awards: literature, physics, chemistry, medicine, and peace Economics was added in 1968, just sixty-seven years after the first awards ceremony Nobel's original legacy of nine million dollars was invested, and the interest on this sum is used for the awards which vary from $30,000 to $125,000 Every year on December 10, the Einniversary of Nobel's death, the awsirds (gold medal, illuminated diploma, and money) are presented to the winners Sometimes politics plays an important role in the judges' decisions Americans have won numerous science awards, but relatively few literature prizes No awards were presented from 1940 to 1942 at the beginning of World War II Some people have won two prizes, but this is rare; others have shared their prizes Question 43: The word "foresaw" in the first paragraph is nearest in meaning to A prevailed B predicted C prevented D postponed Question 44: It is implied that Nobel's profession was in A literature B science C medicine D economics Question 45: The Nobel prize was established in order to A spend money B resolve political differences C honour the inventor of dynamite D recognize worthwhile contributions to humanity Question 46: All of the following statements are true EXCEPT A Politics plays an important role in selecting the winners B Ceremonies are held on December 10 to commemorate Nobel's invention C A few individuals have won two awards D Awards vary in monetary value Question 47: In the first paragraph, "worthwhile" is closest in meaning to A trivial B economic C prestigious D valuable Question 48: In which area have Americans received the most awards? A Peace B Literature C Economics D Science Question 49: The word "legacy" in the second paragraph means most nearly the same as A prize B legend C debt D bequest Question 50: What is the main idea of this passage? A Alfred Nobel created awards in six categories for contributions to humanity B Alfred Nobel made a lasting contribution to humanity C Alfred Nobel left all of his money to science D Alfred Nobel became very rich when he invented dynamite 24 [...]... pronunciation in each of the following questions Question 9:A measure B pressure C leisure D pleasure Question 10:A defended B composed C completed D supported Mark the letter A, B, C, or D on your answer sheet to indicate the word that differs from the other three in the position of primary stress in each of the following questions Question 11:A character B wonderful C publisher D understand Question... Peace B Literature C Economics D Science Question 49: The word "legacy" in the second paragraph means most nearly the same as A prize B legend C debt D bequest Question 50: What is the main idea of this passage? A Alfred Nobel created awards in six categories for contributions to humanity B Alfred Nobel made a lasting contribution to humanity C Alfred Nobel left all of his money to science D Alfred... to dinner next Sunday? D Congratulations! How wonderful! Mark the letter A, B, C, or D on your answer sheet to indicate the word whose underlined part differs from the other three in pronunciation in each of the following questions Question 29:A supported B defended C composed D completed Question 30:A measure B pressure C leisure D pleasure Mark the letter A, B, C, or D on your answer sheet to indicate... Ceremonies are held on December 10 to commemorate Nobel's invention B A few individuals have won two awards C Awards vary in monetary value D Politics plays an important role in selecting the winners Question 30: In which area have Americans received the most awards? A Economics B Literature C Peace D Science Question 31: What is the main idea of this passage? A Alfred Nobel made a lasting contribution... position of primary stress in each of the following questions Question 33:A commitment B collection C enterprise D inflation Question 34:A understand B character C publisher D wonderful Read the following passage and mark the letter A, B, C, or D on your answer sheet to indicate the correct answer to each of the questions from 35 to 41 Before applying to a foreign university, one should consider the... three in the position of primary stress in each of the following questions Question 31:A publisher B understand C wonderful D character Question 32:A collection B inflation C commitment D enterprise Mark the letter A, B, C, or D on your answer sheet to indicate the sentence that is closest in meaning to each of the following questions Question 33: You can refuse to answer my question in no way A In... made a lasting contribution to humanity Question 47: All of the following statements are true EXCEPT A Politics plays an important role in selecting the winners B Awards vary in monetary value C A few individuals have won two awards D Ceremonies are held on December 10 to commemorate Nobel's invention Question 48: It is implied that Nobel's profession was in A medicine B literature C science D economics... Mark the letter A, B, C, or D on your answer sheet to indicate the word that differs from the other three in the position of primary stress in each of the following questions Question 4:A inflation B commitment C enterprise D collection Question 5:A understandB character C wonderful D publisher Mark the letter A, B, C, or D on your answer sheet to indicate the word(s) OPPOSITE in meaning to the underlined... What is the main idea of this passage? A Alfred Nobel left all of his money to science B Alfred Nobel became very rich when he invented dynamite C Alfred Nobel created awards in six categories for contributions to humanity D Alfred Nobel made a lasting contribution to humanity Question 46: In which area have Americans received the most awards? A Economics B Science C Literature D Peace Question 47: The... again One of the (34) about water is distribution Water is not always distributed where the large (35) centres are Some regions get enough rain But it is all in one or two short rainy seasons Question 31:A in B without C for D with Question 32:A characteristics B conditions C situation D quality Question 33:A comes B goes C gets D arrives Question 34:A facts B things C problems D cases Question 35:A ... legend C debt D bequest Question 50: What is the main idea of this passage? A Alfred Nobel created awards in six categories for contributions to humanity B Alfred Nobel made a lasting contribution... B postponed C prevailed D predicted Question 36: What is the main idea of this passage? A Alfred Nobel left all of his money to science B Alfred Nobel made a lasting contribution to humanity... position of primary stress in each of the following questions Question 31:A publisher B understand C wonderful D character Question 32:A collection B inflation C commitment D enterprise Mark the

Ngày đăng: 16/01/2017, 09:53

Từ khóa liên quan

Tài liệu cùng người dùng

Tài liệu liên quan